LSAT Explanation PT 24, S2, Q19: In Australia the population that is

LSAT Question Stem

Each of the statements below, if true, weakens the argument EXCEPT: 

Logical Reasoning Question Type

This is a Weaken question. 

Correct Answer

The correct answer to this question is E. 

LSAT Question Complete Explanation

The question type for this problem is Weaken EXCEPT, which means we are looking for an answer choice that does not weaken the argument. The argument in the passage states that the driving-age population in Australia has grown larger over the last five years, and the annual number of traffic fatalities has declined. The conclusion is that the driving-age population of Australia consists of more skillful drivers now than five years ago.

To help understand this argument, let's use a simple, relatable example. Imagine a school where the number of students has increased over the last five years, but the number of failing grades has decreased. The conclusion would be that the students at the school are more skillful learners now than five years ago.

An "Evaluate" question for this argument could be: "Have there been any changes in teaching methods, school policies, or other factors that could contribute to the decrease in failing grades?"

Now let's analyze the answer choices:

a) This answer choice suggests an alternate cause for the decline in traffic fatalities: a mandatory seat-belt law. This weakens the argument by providing another possible reason for the decline in fatalities, so it is not the correct answer.

b) This answer choice also suggests an alternate cause for the decline in traffic fatalities: a major road repair project. This weakens the argument by providing another possible reason for the decline in fatalities, so it is not the correct answer.

c) This answer choice suggests another alternate cause for the decline in traffic fatalities: an increase in fuel prices, causing Australians to drive less. This weakens the argument by providing another possible reason for the decline in fatalities, so it is not the correct answer.

d) This answer choice suggests yet another alternate cause for the decline in traffic fatalities: an increase in hospital emergency facilities. This weakens the argument by providing another possible reason for the decline in fatalities, so it is not the correct answer.

e) This answer choice strengthens the argument by suggesting that mandatory driver education could have contributed to the increase in skillful drivers. Since it does not weaken the argument (and in fact strengthens it), it is the correct answer for this Weaken Except question.

Previous
Previous

LSAT Explanation PT 25, S2, Q7: Generations of European-history students have been

Next
Next

LSAT Explanation PT 23, S3, Q23: A person's failure to keep a